1answer.
Ask question
Login Signup
Ask question
All categories
  • English
  • Mathematics
  • Social Studies
  • Business
  • History
  • Health
  • Geography
  • Biology
  • Physics
  • Chemistry
  • Computers and Technology
  • Arts
  • World Languages
  • Spanish
  • French
  • German
  • Advanced Placement (AP)
  • SAT
  • Medicine
  • Law
  • Engineering
aniked [119]
2 years ago
7

I dont know this question and i really want to know 51.2+□.85=56.05

Mathematics
2 answers:
hichkok12 [17]2 years ago
8 0

Answer:

51.2+4.85+56.05

Step-by-step explanation:

It would be "4.85"

Hope it helped brainiest plz

stellarik [79]2 years ago
8 0

Answer:

51.2 + 4.85 = 56.05

Step-by-step explanation:

56.05 - 0.85 = 55.2

55.2 - 51.2 = 4

The missing number is 4

If my answer is incorrect, pls correct me!

If you like my answer and explanation, mark me as brainliest!

-Chetan K

You might be interested in
What is the equation of the inverse of the function?<br><br><br> f(x)=x^4+16, x ≥ 0
dangina [55]

Answer:

https://tex.z-dn.net/?f=q%3D0%2C%5C%20r%3D2%2C%5C%20s%3D3%2C%5C%20t%3D-3

7 0
3 years ago
Hello, Brainly community!
ioda

Answer:

(B)  \displaystyle \frac{W(3.1) - W(2.9)}{0.2}

General Formulas and Concepts:

<u>Calculus</u>

Limits

Derivatives

  • The definition of a derivative is the slope of the tangent line.

Derivative Notation

Instantaneous Rates

  • Tangent Line: \displaystyle f'(x) = \frac{f(b) - f(a)}{b - a}

Step-by-step explanation:

Since we are trying to find a <em>rate</em> at which W(t) changes, we must find the <em>derivative</em> at <em>t</em> = 3.

We are given 2 close answer choices that would have the same <em>numerical</em> answer but different <em>meanings</em>:

  1. (A)  \displaystyle  \lim_{t \to 3} W(t)
  2. (B)  \displaystyle \frac{W(3.1) - W(2.9)}{0.2}

If we look at answer choice (A), we see that our units would simply just be volume. It would not have the units of a rate of change. Yes, it may be the closest numerically correct answer, but it does not tell us the <em>rate</em> at which the volume would be changing and it is not a derivative.

If we look at answer choice (B), we see that our units would be cm³/s, and that is most certainly a rate of change. Answer choice (B) is also a <em>derivative</em> at <em>t</em> = 3, and a derivative tells us what <em>rate</em> something is changing.

∴ Answer choice (B) will give us the best estimate for the value of the instantaneous rate of change of W(t) when <em>t</em> = 3.

Topic: AP Calculus AB/BC (Calculus I/I + II)

Unit: Differentiation

Book: College Calculus 10e

8 0
2 years ago
Plz help mee look at the picture thanks
Degger [83]

Answer:

The answer is D for this question. The one at the very bottom of the picture. The other person who answered on this question is incorrect.

Step-by-step explanation:

3 0
3 years ago
Translate this sentence into a multi-step equation Six more than the quotient of a number and 8 is equal to 2.
Korvikt [17]

Answer:

6+n/8=2

Step-by-step explanation:

* More Than = (+) or in some cases (>)

* Quotient of a (#) = Any letter that represents a number, so in this case "n", because we do not know that number yet. We divide this with "8" because it says "and 8".

* Equal = <---

5 0
2 years ago
4) If a = 28 and b = 4, then a/b = ______
exis [7]

Answer:

7

Step-by-step explanation:

You divided both 28 and 4 and you will get 7 as your answer.

7 0
3 years ago
Other questions:
  • 4x-2y=12<br> -4x-9=54<br><br> How can you order a ladder?
    14·1 answer
  • (b) What is 0.36 (six repeating) expressed as a fraction in simplest form?
    11·1 answer
  • A seller buys an item from a manufacturer for 100 and sells it to a customer for 120. Which of these terms describes this extra
    10·1 answer
  • The midpoint of a segment is (2,-5), and one of the endpoints is (3,6). where is the other endpoint?
    15·1 answer
  • Pls help I fell asleep while doing hw and I gotta turn it in at 6 pls help. No false answers.
    8·1 answer
  • I need more answers and I don’t know how to make an account
    10·2 answers
  • The price of a DVD is $24.00 plus 8% sales tax.What is the sales tax on this DVD in dollars and cents?
    8·2 answers
  • Work out the circumference of a circle with diameter 1.8cm.
    8·2 answers
  • PLS HELP ME WITH THIS IM GOING TO LOSE THE YEAR!!
    10·1 answer
  • Which set of ordered pairs could be generated by an exponential function?
    8·1 answer
Add answer
Login
Not registered? Fast signup
Signup
Login Signup
Ask question!